Olimpíadas(Putnam) Inequação Tópico resolvido

Aqui devem ser postados problemas Olímpicos. Informe a olimpíada e o ano no título do tópico. Exemplo: (OBM - 2008).

Moderador: [ Moderadores TTB ]

Avatar do usuário
Autor do Tópico
leomaxwell
3 - Destaque
Mensagens: 564
Registrado em: Ter 11 Jul, 2017 07:30
Última visita: 09-03-19
Jan 2018 10 21:16

(Putnam) Inequação

Mensagem não lida por leomaxwell »

Sejam [tex3]m[/tex3] e [tex3]n[/tex3] inteiros positivos. Mostre que
[tex3]\frac{(m+n)!}{(m+n)^{m+n}}<\frac{m!}{m^m}\frac{n!}{n^n} [/tex3]



All you touch and all you see is all your life will ever be...

Avatar do usuário
alexander4102
iniciante
Mensagens: 6
Registrado em: Dom 04 Fev, 2018 19:59
Última visita: 05-09-18
Fev 2018 07 22:27

Re: (Putnam) Inequação

Mensagem não lida por alexander4102 »

Seja [tex3]E_k = \dfrac{k!}{k^k}[/tex3] . Então, devemos provar a desigualdade
[tex3]E_{m+n} > E_m \cdot E_n[/tex3] .

Note que:
[tex3]\dfrac{E_{k+1}}{E_k} = \dfrac{(k+1)!}{(k+1)^{k+1}} \cdot \dfrac{k^k}{k!} [/tex3]
[tex3]\boxed{\dfrac{E_{k+1}}{E_k} = \left(\dfrac{k}{k+1} \right)^{k}}[/tex3]

Antes de prosseguir, vamos provar o seguinte:
Se [tex3]a[/tex3] e [tex3]b[/tex3] são inteiros positivos, então [tex3]\left(\dfrac{a}{a+1}\right)^{a} > \left( \dfrac{a+b}{a+b+1}\right)^{a+b}[/tex3]
Para isso, basta provar que [tex3]\left(\dfrac{a}{a+1} \right)^{a} > \left( \dfrac{a+1}{a+2}\right)^{a+1}[/tex3] . Feito isso, a demonstração é feita por indução em [tex3]b[/tex3] .

A desigualdade é equivalente a [tex3]\left( \dfrac{a}{a+1}\right)^{a}\left(\dfrac{a+2}{a+1}\right)^{a+1} > 1[/tex3] , que pode ser reescrita como:
[tex3]\left[ \dfrac{a(a+2)}{(a+1)^2} \right]^{a+1} \cdot \dfrac{a+1}{a} > 1[/tex3]
[tex3]\left( \dfrac{a^2 + 2a}{a^2 + 2a + 1}\right)^{a+1}\cdot \dfrac{a+1}{a} > 1[/tex3]
[tex3]\left(1 - \dfrac{1}{(a+1)^2}\right)^{a+1} \cdot \dfrac{a+1}{a} > 1[/tex3]

Para concluir, utilizaremos o que seria uma adaptação da desigualdade de Bernoulli:
Usando a desigualdade das médias para os [tex3]a+1[/tex3] números [tex3]1,1,\dots, 1-\dfrac{1}{(a+1)^2}[/tex3] , todos positivos:

[tex3]\dfrac{1+1 + \dots + \left(1- \dfrac{1}{a+1}\right)}{a+1} \ge \sqrt[a+1]{1-\dfrac{1}{a+1}}[/tex3]
[tex3]1-\dfrac{1}{(a+1)^2} \ge \sqrt[a+1]{1-\dfrac{1}{a+1}}[/tex3]
[tex3]\left(1 - \dfrac{1}{(a+1)^2} \right)^{a+1} > 1 - \dfrac{1}{a+1}[/tex3]
Note que podemos desconsiderar o caso de igualdade, pois [tex3]a>0[/tex3] .

Logo:
[tex3]\left(1 - \dfrac{1}{(a+1)^2}\right)^{a+1} \cdot \dfrac{a+1}{a} > \left(1 - \dfrac{1}{a+1} \right)\cdot \dfrac{a+1}{a} = 1[/tex3]

Voltando a solução do problema:
Faremos indução em [tex3]m,n[/tex3] . O caso [tex3]m=n=1[/tex3] é de fácil verificação. Agora suponha que temos [tex3]E_{m+n} > E_m \cdot E_n[/tex3] para alguns [tex3]m,n[/tex3] inteiros positivos. Então:

[tex3]E_{m+n+1} =E_{m+n} \cdot \left( \dfrac{m+n}{m+n+1}\right)^{m+n}[/tex3]
Pela desigualdade que provamos e hipótese de indução, temos:
[tex3]E_{m+n+1} > \left( \dfrac{m}{m+1}\right)^{m} \cdot E_{m+n}[/tex3]
[tex3]E_{m+n+1} > \left( \dfrac{m}{m+1}\right)^{m} \cdot E_m \cdot E_n[/tex3]
[tex3]E_{m+n+1} > E_{m+1} \cdot E_n[/tex3]

ou

[tex3]E_{m+n+1} > \left( \dfrac{n}{n+1}\right)^{n} \cdot E_{m+n}[/tex3]
[tex3]E_{m+n+1} > \left( \dfrac{n}{n+1}\right)^{n} \cdot E_m \cdot E_n[/tex3]
[tex3]E_{m+n+1} > E_{m} \cdot E_{n+1}[/tex3] .

Dessa forma, provamos que a desigualdade é verdadeira para todo [tex3]m,n[/tex3] inteiros positivos




Responder
  • Tópicos Semelhantes
    Respostas
    Exibições
    Última msg

Voltar para “Olimpíadas”